SLP4

¡Supera tus tareas y exámenes ahora con Quizwiz!

15. As you analyze a child's language sample, you find that she uses many instances of recurrence. Which of the following is an example of recurrence? A. "More juice" B. "No milk" C. "Yummy juice" D. "Want cookie" E. "Mommy's cookie"

A. "More juice"

71. A 74-year-old bilingual Asian man has had a stroke, and you are seeing him for therapy. He is recovering both his primary language and his English skills, but you are working only in English. There are no interpreters available, unfortunately, and the family has indicated that they would prefer treatment to be conducted in English since many of the patient's grandchildren speak English fluently. Which one of the following productions would be an example, on the patient's part, of English influenced by his primary language and not the stroke? A. "She going over there now." B. "I not done got eat breakfast dinner." C. "She not have no dollar in her purse." D. "They be havin' many fun." E. "I see cow brown in picture there over."

A. "She going over there now."

111. ASHA (American Speech-Language-Hearing Association) regulates professional practice in speech-language pathology and audiology by specifying the scope of practice. According to the existing scope-of-practice documents, which of the following statements is false? A. ASHA does not allow infant feeding by speech-language pathologists (SLPs). B. Memory and such other cognitive aspects of communication disorders are within the SLP's scope of practice. C. SLPs should not treat esophageal-phase swallowing disorders. D. Counseling is not just the province of psychologists; SLPs, too, can counsel their clients. E. Sensory awareness training related to swallowing disorders is within the SLP's scope of practice.

A. ASHA does not allow infant feeding by speech-language pathologists (SLPs).

29. Adolescent language disorders may have slightly different characteristics than do the same disorders in younger children. If you were asked to assess a senior high school student, what special positive or negative characteristics (among others not mentioned) would you look for? A. Agreement problems, limited use of low-frequency structures, lack of tact, maze behavior, and difficulty modifying his or her own statements B. Marked morphologic deficiencies but good peer-group slang usage C. Minimal or no dysfluencies and absence of false starts because of impulsive expressions D. In spite of limited language skills, correct usage of register E. No particular difficulty in using subordinate clauses

A. Agreement problems, limited use of low-frequency structures, lack of tact, maze behavior, and difficulty modifying his or her own statements

119. A clinician in a private practice is approached by the parents of Cole, a 5-year-old boy. They want to place Cole in kindergarten in the fall, but they share that "we know there's something wrong with him—we're just not sure what." According to Cole's parents, he is a "sweet, lovable boy who will go to anybody. He likes to sing a lot, too." Because the parents live in a rural area, health-care access has been limited. After seeing Cole for the first time, the clinician refers his parents to a neurologist because she suspects that Cole has a syndrome. Cole is small for his age and has an elfin-like appearance characterized by a small chin, turned-up nose, puffiness around the eyes, a long upper lip, and a wide mouth. Cole's teeth are small and widely spaced. This syndrome is caused by a rare genetic disorder that affects an estimated 1 out of every 20,000 babies. It is caused by what? A. An abnormality on chromosome 7, including a gene that makes the protein elastin B. Autosomal dominant inheritance and deletion in the region of the long arm of chromosome 15 (15q11-13) C. A spontaneous autosomal dominant mutation whose gene and locus is FGR2 at 10q25-26 D. A missing part of chromosome 23, known as 23q11 E. An expanded number of cytosine-guanine-guanine (CGG) nucleic acid repeats on a specific gene on one of the distal ends of the X chromosome

A. An abnormality on chromosome 7, including a gene that makes the protein elastin

25. Your assessment of a child with cerebral palsy (CP) shows that in addition to significant speech and language problems, the child exhibits slow, writhing involuntary movements. Your correctly diagnose that the child has which of the following? A. Athetoid CP B. Ataxic CP C. Quadriplegia D. Spastic CP E. Diplegia

A. Athetoid CP

76. Popular forms of amplification today include hearing aids and cochlear implants. Which one of the following is not true about these devices? A. Cochlear implants deliver amplified sound to the ear canal, while hearing aids deliver electrical impulses (converted from sound) directly to the auditory system. B. Cochlear implants can help prelingual children to make substantial progress toward maximizing their potential. C. Cochlear implants may be used with children and some adults who have sensorineural hearing loss. D. Digital hearing aids provide a better signal-to-noise ratio than do analog aids. E. A consideration in fitting clients with hearing aids is whether the clients are motivated to use and properly care for the aids.

A. Cochlear implants deliver amplified sound to the ear canal, while hearing aids deliver electrical impulses (converted from sound) directly to the auditory system.

21. As you evaluate the language of an eighth-grade boy, Derek, and listen to him talk about his hobbies and interests, he says things such as "I like to play football, and I also like Mario Cart Wii" and "My football team won the championship last Saturday; later, we celebrated at a pizza place." What has Derek just used? A. Compound sentences containing two independent clauses B. Complex sentences containing two independent clauses C. A compound and a complex sentence D. Complex sentences containing two dependent clauses E. Complex sentences containing an independent and a dependent clause

A. Compound sentences containing two independent clauses

114. As a clinician in a medically based private practice, you receive a referral of 37-year-old Jan, who has been a physical education teacher for the last 15 years. Jan works part time as a telemarketer, and, according to her husband, she "is glued to her cell phone." Jan also sings in the church choir. She has been hoarse for several years and tells you during the case history that "I've ignored the way I sound—it's just me. I haven't felt like I've needed to change anything." However, Jan shares that lately she has been feeling a lot of pain, and the hoarseness is substantially worse. She says, "Sometimes when I talk, it's almost like there's a 'double voice.'" Jan tells you she is worried because her job depends upon being able to yell and raise her voice on the playground as she teaches physical education. She is worried that she might lose her job if she can no longer yell. You immediately refer Jan to an otolaryngologist for a thorough examination of her vocal folds. You then proceed to do your own perceptual and instrumental evaluation. You arrive at a number of findings, including the fact that Jan has increased laryngeal dysphonia, airway resistance, and a maximum phonation time of 7 seconds. You think that Jan is a probable candidate for phonosurgery, but you will wait for the otolaryngologist's diagnosis and recommendations. When you assess Jan, you indeed find the presence of "double voice." What is the perception of two distinct simultaneous pitches during phonation? A. Diplophonia B. Strain-strangle C. A possible indicator of unilateral vocal fold paralysis D. Cul-de-sac resonance E. A symptom of adductor spasmodic dysphonia

A. Diplophonia

36. You are working with Devin, a 9-year-old boy with an orofacial myofunctional disorder (OMD). Devin has been referred by his orthodontist because orthodontia is not effective due to Devin's problems accompanying the OMD. You have been asked to evaluate Devin and provide remediation, if needed. Which one of the following most accurately describes the situation you will probably encounter? A. During Devin's speech production, his tongue may exert some force against his front teeth and he will probably have a deviant swallow as well as articulation errors involving /s/, /z/, and others. B. Devin will probably have a posterior open bite, no difficulties with swallowing, and problems producing /b/ and /v/. C. Devin will probably have an anterior open bite and a deviant swallow but normal posture of his tongue at rest. D. Devin will probably have a deviant swallow, articulation errors of /s/ and /z/, and an anterior open bite, but it is not within the purview of the speech-language pathologist to provide intervention; you can only provide an evaluation. E. When Devin swallows, he will typically place his tongue tip behind his alveolar ridge, and the body of his tongue will push the fluid or solid posteriorly for swallowing.

A. During Devin's speech production, his tongue may exert some force against his front teeth and he will probably have a deviant swallow as well as articulation errors involving /s/, /z/, and others.

72. A teacher has referred a fifth-grade girl, Phaiwanh, to you for a speech-language assessment. The teacher is concerned because she feels that Phaiwanh is academically "behind her peers." Phaiwanh and her family are Laotian refugees, and they have been in the United States for 8 months. Because Phaiwanh has been in refugee camps most of her life, her schooling in Laos was quite limited. Her parents tell you they estimate that she has had approximately 1.5 years of schooling in Laos. The teacher is concerned that Phaiwanh may have an underlying language-learning disability, and she wonders if Phaiwanh is eligible for speech-language services. What would be the best combination of assessment techniques to use when you evaluate Phaiwanh? A. Dynamic assessment, language samples in Laotian, and observations of her interaction with family members and other Laotian children B. Use of formal, standardized, norm-referenced tests in English combined with observations of Phaiwanh's interactions in Laotian with peers, family members, and adults from the local community C. Use of a district-developed test for Laotian students in your geographic area and administration of questionnaires to Phaiwanh's teachers and family D. Use of school records of Phaiwanh's achievement and performance so far in the English-speaking classroom and use of the Language Processing Test translated into Laotian by an interpreter E. Dynamic assessment, language samples in Laotian, and use of the Peabody Picture Vocabulary Test-Fourth Edition and Test of Language Development-Fourth Edition, translated into Laotian

A. Dynamic assessment, language samples in Laotian, and observations of her interaction with family members and other Laotian children

13. The concept of period is related to which of the following? A. Frequency of sound vibrations B. Amplitude of sound vibrations C. The force with which the sound vibrates D. The pressure exerted by sound waves E. The density of the medium that transmits sound

A. Frequency of sound vibrations

97. A clinician in private practice receives many referrals from local public school clinicians. The children being referred are those whose language test scores are too high to legally qualify them for therapy in the public schools. The parents of these children want services and are willing and able to pay for them out of pocket. The clinician in private practice often administers the Test of Word Intentionality, Temporality, Tenses, and Rules (TWITTER). One day, she becomes curious about the "typical score" of the children to whom she administers the TWITTER. The clinician takes the TWITTER scores of 21 children and lines them up in order from highest to lowest score: 33 46 48 51 55 60 69 73 73 73 73 82 85 89 91 93 95 95 95 96 98 The clinician concludes that the "typical" or average score on the TWITTER is 73. In terms of measures of central tendency, what is the average score calculated by the clinician? A. Mode B. Measure of central variability C. Median D. Standard deviation E. Mean

A. Mode

65. While working in a hospital setting, you are asked to evaluate a 70-year-old patient who has had a brainstem stroke. The medical records indicate that the patient has difficulty swallowing. When you conduct test swallow trials, you observe anterior tongue movements, food residue in the anterior and lateral sulcus, premature swallow, and reduced range of tongue elevation. Which of the following is the most likely diagnosis you would make on this patient? A. Predominantly a disorder of the oral phase B. Predominantly a disorder of the pharyngeal phase C. Predominantly a disorder of mastication D. Predominantly a problem of delayed or absent swallowing reflex E. Predominantly a problem of a tracheoesophageal fistula

A. Predominantly a disorder of the oral phase

47. In treating a preschool child who stutters, you have several options. You select the response cost method, which includes which of the following steps? A. Present a token for each fluent production, and withdraw a token for each dysfluency. B. Reinforce all fluent productions with a token, and ignore all dysfluencies. C. Reinforce all fluent productions with a token, and ask the child to pause after each dysfluency and continue speech after the pause. D. Give a stack of tokens at the very beginning of the session and take a token back for each dysfluency. E. Reinforce fluent productions with a token, but use verbal corrective feedback for dysfluencies.

A. Present a token for each fluent production, and withdraw a token for each dysfluency.

34. You are a clinician in a school district where increasing numbers of children are being diagnosed with childhood apraxia of speech (CAS). As you plan interventions for these children, what can you anticipate they will display? A. Slow, effortful speech; inconsistent and multiple articulation errors; and more difficulty with consonant clusters than fricatives B. Consistent sound errors accompanied by prolongation of speech sounds, voicing and devoicing errors, and deviations in prosody C. Deficits in phonological representation, problems with hyponasality, and rapid speech D. A limited sound inventory, unusual errors of articulation (e.g., metathesis), and rapid speech E. The most difficulty with affricates, followed by consonant clusters and stops, and then by slow, effortful speech

A. Slow, effortful speech; inconsistent and multiple articulation errors; and more difficulty with consonant clusters than fricatives

60. You have been referred a 75-year-old woman by a neurologist who suspects hypokinetic dysarthria in her and has requested a speech evaluation and diagnosis of her communication difficulties. To confirm a diagnosis of hypokinetic dysarthria, you will be especially alert to which of the following? A. Symptoms of Parkinson's, Alzheimer's, or Pick's disease; evidence of damage to the basal ganglia; mask-like face; slowness of movement; micrographic writing; monopitch; and imprecise articulation, among other symptoms B. Increased tone and rigidity of muscles, symptoms of myoclonus, dystonia, voice tremors, intermittent hypernasality, excessive loudness variations, and forced and sudden inspiration, among other symptoms C. Bilateral facial weakness with less-severe lower face weakness, perhaps normal jaw strength, hyperactive gag reflex, hyperadduction of vocal folds, short phrases, continuous breathy voice, and predominant hypernasality, among other symptoms D. Fasciculations and fibrillations of muscles, progressive muscle weakness with use and recovery with rest, nasal emissions, harsh voice, monopitch, and monoloudness, among other symptoms E. Evidence of cerebellar damage; rotated or tilted head posture; over- and undershooting of movement targets; jerky, inaccurate, and halting movement; prolonged phonemes and intervals between words or syllables; and impression of drunken speech, among other symptoms

A. Symptoms of Parkinson's, Alzheimer's, or Pick's disease; evidence of damage to the basal ganglia; mask-like face; slowness of movement; micrographic writing; monopitch; and imprecise articulation, among other symptoms

1. The intercostal muscles are in between the ribs and play an important role in respiration. The two sets of intercostals perform different functions. Select the correct statement. A. The 11 paired internal intercostal muscles pull the ribs downward to decrease the diameter of the thoracic cavity for exhalation. B. The 11 paired external intercostals pull the ribs downward to decrease the diameter of the thoracic cavity for exhalation. C. The 10 paired internal intercostals raise the ribs up and out to increase the diameter of the thoracic cavity for exhalation. D. The 10 paired external intercostals pull the ribs down to decrease the diameter of the thoracic activity for inhalation. E. The 12 paired internal and external intercostals raise and lower the ribs to increase and decrease the diameter of the thoracic cavity.

A. The 11 paired internal intercostal muscles pull the ribs downward to decrease the diameter of the thoracic cavity for exhalation.

109. A clinician is a member of a cleft palate and craniofacial team, which asks her to conduct an objective assessment of a 6-year-old child's velopharyngeal mechanism. The clinician decides to do nasopharyngoscopy, where the nasopharyngoscope is passed through the middle meatus and back to the area of velopharyngeal closure. What will this procedure enable the clinician to observe? A. The child's posterior and lateral pharyngeal walls, as well as the nasal aspect of the velum and the adenoid pad as the child produces sentences B. The child's posterior and lateral pharyngeal walls, as well as the adenoid pad as the child sustains /a/ C. The child's nasal aspect of the velum and the adenoid pad as the child produces CVC words D. The child's posterior and lateral pharyngeal walls, as well as the nasal aspect of the velum and the adenoid pad as the child produces nasals /n/ and /m/ E. The child's adenoid pads and anterior pharyngeal walls as the child prolongs /s/

A. The child's posterior and lateral pharyngeal walls, as well as the nasal aspect of the velum and the adenoid pad as the child produces sentences

99. You give a new test of cognition to patients on your hospital caseload who have been diagnosed with Alzheimer's disease. You find a vast spread in the scores of the patients, depending upon how advanced their Alzheimer's disease is. You want to determine the range of scores of your patients. What can you calculate to do this? A. The difference between the highest and the lowest scores of the patients to whom the test has been administered B. The lowest and highest 25% of the scores C. The middle 50% of the scores D. The middle 50% of the scores divided by 2 E. The variance plus the difference between the highest and lowest scores

A. The difference between the highest and the lowest scores of the patients to whom the test has been administered

90. You are assessing an African American child with a suspected language disorder. You have prepared a plan of assessment in which you will assess and teach a few skills before you reassess the child to make a final diagnosis. What is this approach called? A. The dynamic assessment B. The criterion-referenced approach C. The authentic approach D. The portfolio approach E. The correlation approach

A. The dynamic assessment

40. To diagnose stuttering, the clinician needs a definition of it. Although there are different definitions of stuttering, a majority of them consider forms of dysfluencies. Which of the following is a definition of stuttering that includes dysfluencies? A. The one offered by Van Riper B. The one offered by Johnson C. The one offered by psychoanalysts D. The one offered by Sheehan E. The one based on the stuttering "moment"

A. The one offered by Van Riper

9. Consonants and vowels have certain distinct characteristics. Of the following statements, which one accurately distinguishes the two classes of speech sounds? A. Vowels form the nucleus of syllables, whereas consonants release and arrest syllables. B. Consonants are syllabic, whereas vowels cannot stand alone to form syllables. C. The vocal tract is constricted for vowel productions, whereas it is open for consonantal productions. D. Consonants may be described in terms of distinctive features, whereas vowels cannot be so described. E. Airflow is continuous for consonants, whereas the flow is constricted for vowels.

A. Vowels form the nucleus of syllables, whereas consonants release and arrest syllables.

39. You find that a child you have evaluated uses a number of phonological processes. One of those processes is stopping. You know this when you hear the child make what substitutions? A. du/zoo B. kown/clown C. bus/bush D. wing/ring E. tuhree/three

A. du/zoo

85. In spite of their significant disadvantages, standardized tests are used by most clinicians in assessing clients because such tests A. help qualify children for clinical services in public schools B. are required by federal laws C. help generate the most reliable and valid assessment data D. help select target behaviors for treatment E. help track progress in treatment

A. help qualify children for clinical services in public schools

5. One major distinction between the pyramidal and the extrapyramidal systems is that A. the pyramidal system controls voluntary and fine-motor movements, whereas the extrapyramidal system controls the postural support for fine-motor movement B. unlike the pyramidal system, the extrapyramidal system has direct connections with lower motor neurons C. the pyramidal system is an indirect activation system, whereas the extrapyramidal system is a direct activation system D. the pyramidal system is important for speech production, but the extrapyramidal system plays no role in this activity E. only the fibers of the pyramidal tracts originate in the cortex

A. the pyramidal system controls voluntary and fine-motor movements, whereas the extrapyramidal system controls the postural support for fine-motor movement

50. Sometimes specialists assess the lung volume of voice patients because breath support is inadequate. Specialists can measure __________, or the total volume of air in the lungs; other measurements can include __________, or the amount of air inhaled and exhaled during a normal breathing cycle; and __________, or the volume of air that the patient can exhale after a maximal inhalation. A. total lung capacity, tidal volume, vital capacity B. vital capacity, tidal capacity, total lung volume C. vital capacity, total lung capacity, tidal volume D. tidal volume, total lung capacity, vital volume E. vital capacity, total lung volume, tidal volume

A. total lung capacity, tidal volume, vital capacity

22. You have completed your assessment of a 6-year-old boy's language skills. Reports from such specialists as an audiologist, a pediatrician, and a psychologist are negative, but your assessment results show that the boy's language skills are limited, with marked deficiencies in producing grammatical morphemes. The correct diagnosis you convey to the parents of this child is A. "Your child has a language disorder due to lack of stimulation during infancy." B. "Your child has a specific language impairment." C. "Your child has a central auditory processing disorder that causes language problems." D. "Your child has a language disorder that is due to subtle neurological impairment." E. "Your child has an articulation disorder because he has an overbite."

B. "Your child has a specific language impairment."

19. A child being assessed for a possible language impairment says to you, "My dad put my shoes on my feet before he drove me to school." This sentence has A. 14 words, 14 morphemes B. 14 words, 15 morphemes C. 14 words, 16 morphemes D. 14 words, 17 morphemes E. 14 words, 18 morphemes

B. 14 words, 15 morphemes

45. You are developing a treatment program for a 30-year-old man with a long history of stuttering. Your brief trial therapy suggests that he could benefit from a fluency shaping procedure. You are now developing specific treatment targets that you would teach. What would those targets be, and in what order would they be taught? A. Gentle phonatory onset, syllable prolongation, and airflow management, taught in that order B. Airflow management, gentle phonatory onset, and syllable prolongation, taught in that order C. A slow rate of speech achieved through pauses between words, airflow management, and gentle phonatory onset, taught in that order D. Airflow management, cancellation, pull-outs, and preparatory sets, taught in that order E. Desensitization, stuttering modification, and psychological conflict resolution, taught in that order

B. Airflow management, gentle phonatory onset, and syllable prolongation, taught in that order

30. In the mand-model method of teaching language skills to children, the clinician does which of the following? A. Prompts an elaboration of a verbal response, and the child is required to imitate B. Asks questions like "Tell me what you want" and models or prompts the correct response if necessary C. Adds comments to the child's production, but there is no response requirement D. Repeatedly and consecutively models a particular structure but does not require the child to imitate it E. Models elaborated verbal responses, but the child is not required to imitate

B. Asks questions like "Tell me what you want" and models or prompts the correct response if necessary

67. Tarandeep is a 6-year-old girl in an all-English-speaking first-grade classroom. Tarandeep's parents emigrated from India 8 years ago, and Tarandeep was born in the United States. She came to an all-English kindergarten speaking only Urdu; kindergarten was her first regular exposure to English. Tarandeep had no prior preschool experience in either Urdu or English. The first-grade teacher has referred Tarandeep for a speech-language evaluation because he says that although she interacts well with her English-speaking classmates on the playground and can "carry on a decent conversation," she is "behind" her classmates in written-language skills (e.g., spelling, reading). He thinks Tarandeep has good potential to do well in math and science, but states she is "lagging behind her friends" in these areas as well. Based on Tarandeep's background, what can you conclude? A. Because cognitive-academic language proficiency (CALP) takes approximately 5 to 7 years to develop to a native-like level, it can be expected that Tarandeep will lag somewhat behind English-speaking peers in written language, math, and science skills. B. Because basic interpersonal communication skills (BICs) take approximately 2 years to develop to a native-like level, Tarandeep is developing on an appropriate time line. C. Because Tarandeep has been in an all-English-speaking classroom setting for at least a year, her written language, math, and science skills should be more developed than they are. Her difficulties are a red flag, and a speech-language assessment should be conducted. D. Tarandeep needs to be tested by the psychologist because she probably has mental retardation. E. There is a good possibility that Tarandeep has emotional difficulties, and she should be referred to a social worker.

B. Because basic interpersonal communication skills (BICs) take approximately 2 years to develop to a native-like level, Tarandeep is developing on an appropriate time line.

52. You are asked to see a 6-year-old boy, Gabe, for potential therapy because he is very hoarse. Gabe has been hoarse for approximately 7 months. He is an active, happy first grader who loves sports and is engaged in various types of sports year-round (e.g. soccer, baseball). Reportedly, Gabe frequently screams at games. At the school, there is one 15-minute recess in the morning and one 30-minute recess after lunch. You observe Gabe on the playground at recess several times over a period of 2 weeks and see that he loves to run, play, and yell loudly with his friends. Gabe's parents have given you a letter from the ENT that definitively states that Gabe has vocal nodules. After an evaluation, what is the first thing you would do? A. Send Gabe to a counselor to probe into any possible psychological or emotional factors that are contributing to his vocally abusive behavior B. Focus on identification and reduction of vocally abusive behavior such as yelling and screaming; use computer games and prizes to help motivate Gabe to use better vocal habits C. Monitor Gabe's vocal status by seeing him once every 3 months for the next year to observe whether his hoarseness gets better or worse D. Prescribe 3 or 4 weeks of almost total voice rest, telling Gabe and his parents that he can speak only when he absolutely has to—no yelling at recess or when he plays sports E. Give Gabe and his parents reading materials that discuss vocal abuse, and tell the parents that if the hoarseness does not resolve in 2 or 3 months, you will probably see Gabe for voice therapy on a weekly basis

B. Focus on identification and reduction of vocally abusive behavior such as yelling and screaming; use computer games and prizes to help motivate Gabe to use better vocal habits

61. You have completed the case history and assessment of a 72-year-old man and have found, among other things, uninhibited and inappropriate social behavior, excessive eating, depression, impaired judgment, and dominant language problems, with better-preserved memory and orientation. The neurologist's report suggests dense intracellular formation in the neuronal cytoplasm. Your most likely diagnosis would be which of the following? A. Dementia of the Alzheimer's type B. Frontotemporal dementia associated with Pick's disease C. Right hemisphere syndrome D. Dementia associated with Parkinson's disease E. Dementia due to Creutzfeldt-Jakob disease

B. Frontotemporal dementia associated with Pick's disease

112. Both ASHA and state governments regulate the practice of speech-language pathology. Clinicians often have to meet different requirements for different professional settings. Among the following choices, which statement is incorrect? A. In many states, it is not essential to get the ASHA certification to work as a clinician in public schools. B. In most states, both the state licensure and ASHA certification are required to work as an SLP in public schools. C. Although widely recognized by employers, the ASHA certifications do not have the legal authority of state licensures. D. A state education department's credential is not the same as the same state's licensure. E. Most state licensure requirements are automatically met when a graduate student meets the ASHA standards for certification.

B. In most states, both the state licensure and ASHA certification are required to work as an SLP in public schools.

56. Select the correct statement. A. It is not possible to have Broca's aphasia without a specific injury to Broca's area. B. Injury to Broca's area is not essential to have Broca's aphasia. C. Auditory comprehension is typically worse than speech production in Broca's aphasia. D. Syntactic skills are severely impaired in anomic aphasia. E. Repetition skills are typically impaired in transcortical motor aphasia.

B. Injury to Broca's area is not essential to have Broca's aphasia.

26. Among the following symptoms, which one is not an especially significant feature of traumatic brain injury in children? A. Sentence comprehension problems B. Marked deficiency in producing grammatical morphemes C. Topic maintenance D. Word-retrieval problems E. Hyperactivity, lethargy, or withdrawal

B. Marked deficiency in producing grammatical morphemes

31. A mother brings her 4-year-old child, Aubrey, to you for an evaluation. She says that at preschool, children have difficulty understanding what Aubrey is saying. The preschool teachers are concerned because they think Aubrey "is behind in her pronunciation." Your evaluation reveals that Aubrey does indeed need intervention, and you decide the most appropriate approach is a type of therapy based on metalinguistic awareness. In this therapy approach, you will help Aubrey improve her acquisition of rules of the phonological system. What will you be using? A. McCabe and Bradley's multiple phoneme approach B. Metaphon therapy C. McDonald's sensory-motor approach D. Baker and Ryan's Monterey Articulation Program E. Van Riper's approach

B. Metaphon therapy

98. A speech-language pathologist on a cleft palate and craniofacial team wishes to develop a simple measure of hypernasality to begin quantifying (however subjectively) the amount of hypernasality he hears in the speech of the children seen by the team each month. He will pass on this information to the plastic surgeon and other team members to assist them in making surgical decisions for each child. The speech-language pathologist devises the scale shown below: 1 2 3 4 5 almost no hypernasality slighthypernasality moderate hypernasality great amount of hypernasality What is this type of scale called? A. Logarithmic scale B. Ordinal scale C. Nominal scale D. Interval scale E. Ratio scale

B. Ordinal scale

78. A mother brings her 4-year-old daughter, Megan, for a hearing screening. She shares that Megan has had many middle ear infections, which have been treated with antibiotics. These middle ear infections started when Megan was 2 years old. The mother has been told by a friend that "middle ear tubes" might be a good option for Megan, but the mother states that she is afraid of the surgery that this would entail. After all, Megan is only 4 years old, and her mother does not want her to be traumatized. The mother tells you: "Even though Megan ignores me sometimes, I think everything will be OK. I just wanted her to get checked out to be sure." The audiologist assesses Megan, with the resulting audiogram shown below. What type of problem does Megan have based on this audiogram? A. Mixed loss B. Otitis media with effusion C. Sensorineural loss D. Noise-induced loss E. Otosclerosis

B. Otitis media with effusion

49. You are working in a hospital. A patient, Mr. M., has been referred to you because he is having difficulty adducting his vocal folds. He had surgery for thyroid problems, during which he sustained damage to another structure. Because Mr. M. has especially been having difficulty with vocal-fold adduction, to what might you suspect that he had damage? A. Ventricular vocal folds B. Recurrent laryngeal nerve C. Superior laryngeal nerve D. External carotid artery E. Thyroid cartilage

B. Recurrent laryngeal nerve

115. As a clinician in a medically based private practice, you receive a referral of 37-year-old Jan, who has been a physical education teacher for the last 15 years. Jan works part time as a telemarketer, and, according to her husband, she "is glued to her cell phone." Jan also sings in the church choir. She has been hoarse for several years and tells you during the case history that "I've ignored the way I sound—it's just me. I haven't felt like I've needed to change anything." However, Jan shares that lately she has been feeling a lot of pain, and the hoarseness is substantially worse. She says, "Sometimes when I talk, it's almost like there's a 'double voice.'" Jan tells you she is worried because her job depends upon being able to yell and raise her voice on the playground as she teaches physical education. She is worried that she might lose her job if she can no longer yell. You immediately refer Jan to an otolaryngologist for a thorough examination of her vocal folds. You then proceed to do your own perceptual and instrumental evaluation. You arrive at a number of findings, including the fact that Jan has increased laryngeal dysphonia, airway resistance, and a maximum phonation time of 7 seconds. You think that Jan is a probable candidate for phonosurgery, but you will wait for the otolaryngologist's diagnosis and recommendations. You need to view Jan's vocal folds. To achieve the most precise diagnostic results, you decide to employ a method that uses a pulsing light to permit the optical illusion of slow-motion viewing of the vocal folds. What is this method called? A. Electroglottography B. Stroboscopy C. Electromyography D. Videofluoroscopy E. Nasoendoscopy

B. Stroboscopy

87. You have just administered a standardized test of articulation to a boy with speech sound disorders. You had decided that the test was appropriate for the child because children of the same background as the child were sampled in the normative standardization process. The child scored in the 50th percentile on the test. Which of the following statements is a correct interpretation of this score? A. The child is below average in speech sound production. B. The child's score is equivalent to the median in the normative sample. C.The child is above average in speech sound production. D. The test results suggest target speech productions for treatment. E. The test may be readministered to assess progress in treatment.

B. The child's score is equivalent to the median in the normative sample.

11. Being a branch of physics, acoustics describes various physical properties of sound, its generation, and propagation. Of the following descriptions, which one is incorrect? A. Newton's law of inertia states that all bodies always remain at rest until a force sets them into motion. B. The process of a sound wave traveling back after hitting an obstacle with no change in its speed is called refraction. C. Formant frequency is a frequency region with concentrated energy. D. Measured in terms of Nt, force is the product of mass and acceleration. E. A sinusoidal wave contains a single frequency.

B. The process of a sound wave traveling back after hitting an obstacle with no change in its speed is called refraction.

3. Intrinsic muscles of the larynx serve important functions in phonation. Some are primarily vibrating muscles, whereas others, by their actions, affect the actions of the vocal folds and the resulting quality of phonation. Of the statements that follow, select the one that is incorrect. A. The thyroarytenoids are divided into two muscle masses. B. The transverse arytenoids are laryngeal abductors. C. Intrinsic laryngeal muscles are primarily responsible for controlling sound production. D. The internal thyroarytenoids are also known as the vocalis muscle. E. The cricothyroid muscle lengthens and tenses the vocal folds.

B. The transverse arytenoids are laryngeal abductors.

102. A hospital-based speech-language pathologist is serving increasing numbers of patients with Parkinson's disease. This clinician, Jason, works to help his patients increase their intelligibility. One way that Jason evaluates the success of the therapy he provides for these patients is to rate their overall intelligibility of speech before and after they participate in 10 weeks of articulation and voice therapy. He finds that the patients appreciate these before-and-after measures of their progress. However, after seeing a number of patients with Parkinson's disease, Jason realizes that a potential problem with his before-and-after intelligibility ratings is that he has become accustomed to the patients' speech as he has gotten to know them, and this could be affecting the "after" intelligibility rating. Jason goes to a local university and selects four speech-language pathology graduate students to watch before-and-after videos of his patients and independently rate each patient's intelligibility. He finds that, happily, the four students' ratings of the same patients vary little. For example, Student A rates Patient 1 as "90% intelligible after 10 weeks of therapy;" Student B rates Patient 1 as "92% intelligible," and Students C and D rate this same patient as "88% intelligible." In this situation, what can one conclude? A. There is low interjudge reliability. B. There is high interjudge reliability. C. There is moderate interjudge reliability. D. There is low intrajudge reliability. E. There is high intrajudge reliability.

B. There is high interjudge reliability.

108. A child from Culber City transfers to Central City, and his file indicates that he had been receiving speech-language services in Culber City. Some pages of the report from the speech-language pathologist in Culber City are missing. However, the first page indicates that this child has Moebius syndrome. He also has a history of frequent hospitalizations. What can the Central City clinician probably expect to find? A. This child has syndactyly, cranial synostosis resulting in smaller anterior-posterior skull diameter, high forehead, an arched and grooved hard palate, class III malocclusion, and possible cleft of the hard palate. B. This child has delayed language and an articulation disorder as well as bilabial paresis and weak tongue control for lateralization, elevation, depression, and protrusion; a mask-like face; a history of feeding problems in infancy; and unilateral or bilateral paralysis of the abductors of the eye. C. This child has a small maxillary structure, sphenoethmoidal synchondroses, ocular hypertelorism, facial asymmetry including a tall forehead, and brachycephaly. D. This child has low muscle tone, a history of early feeding difficulties, initial failure to thrive, obesity after the first year, and underdeveloped genitals. E. This child has underdeveloped facial bones, including mandibular hypoplasia, malar hypoplasia, dental malocclusion, and downwardly slanted palpebral fissures.

B. This child has delayed language and an articulation disorder as well as bilabial paresis and weak tongue control for lateralization, elevation, depression, and protrusion; a mask-like face; a history of feeding problems in infancy; and unilateral or bilateral paralysis of the abductors of the eye.

74. A teacher refers Juan to you for a speech-language evaluation. Juan, a second grader from the Dominican Republic who speaks both Spanish and English with equal fluency, transferred to your school district 3 months ago from another district in your state. In his previous district, Juan was in a bilingual classroom, where his primary language of Spanish was maintained, and he was also exposed to English. According to Juan's report card from the previous district, "Juan does well speaking both Spanish and English. I [the teacher] think he is beginning to show a preference for English. Juan is performing adequately in all academic areas." The second-grade teacher at your school, who teaches only in English, feels that after 3 months in her classroom, "Juan is catching on slowly. I wonder if he needs special education. It seems that he would benefit from speech therapy." What would be your best course of action? A. Ask the district to send you a bilingual, Spanish-speaking, speech-language pathologist who can evaluate Juan, because you are sure that he has a language disorder B. Use a dynamic-assessment model to evaluate Juan's language-learning ability, and combine this with classroom observations over the next 2 to 3 months to evaluate his progress C. Use a variety of English screening instruments to screen Juan's English ability because these instruments are ecologically valid for Juan D. Do nothing at present and tell the teacher that you will wait for 6 months to see how Juan progresses in her classroom E. Ask Juan's parents to sign a permission form so he may be assessed immediately in English, using only English tests, since English is apparently beginning to be Juan's preferred language

B. Use a dynamic-assessment model to evaluate Juan's language-learning ability, and combine this with classroom observations over the next 2 to 3 months to evaluate his progress

18. You are working with Chad, a 10-year-old child who has a language impairment. The teacher tells you that other students get angry with Chad because he frequently says things such as "Gimme that pencil" and "Move your books over." As a clinician, what will you focus on in therapy to help Chad get along better with others and have improved pragmatic skills? A. Use of gerunds B. Use of indirect requests C. Use of complex sentences D. Use of abstract adjectives E. Use of derivational morphemes

B. Use of indirect requests

82. Which one of the following is a homophenous pair? A. ship-rip B. bean-mean C. pine-vine D. honey-money E. let-vet

B. bean-mean

93. You are working with a mother who wishes to reduce her son's whining behavior. You have determined that because of his limited oral language skills, he whines to get what he wants. In the clinic, you have established a few mands the child can reliably produce (e.g., "I want [juice] [water] [candy] [cookie] [car]" and so forth). Now you are asking the mother not to respond to whining, but instead to prompt the verbal mands and immediately give what the child mands. The mother reports success with the procedure. This procedure is known as' A. differential reinforcement of incompatible behaviors (DRI) B. differential reinforcement of alternative behaviors (DRA) C. differential reinforcement of low rates of responding (DRL) D. differential reinforcement of other behavior (DRO) E. negative reinforcement of manding

B. differential reinforcement of alternative behaviors (DRA)

64. The description of the different phases of normal swallow suggest that A. swallowing consists of a series of discrete actions described in terms of the oral preparatory, oral, pharyngeal, and esophageal phases. B. in spite of being analyzed in terms of phases, swallowing typically is a continuous process. C. the oral preparatory phase is not linked to the oral phase. D. the pharyngeal phase, because it consists of reflex actions, is not closely related to the oral phase. E. the esophageal phase is unrelated to the other phases because it is not under voluntary control.

B. in spite of being analyzed in terms of phases, swallowing typically is a continuous process.

70. Rosalia is a third-grade Mexican American Spanish-speaking 8-year-old girl who is in the process of learning English. Her parents emigrated from Mexico 2 years ago; thus, Rosalia was exposed first to Spanish at home and to English in first grade, when she was 6 years old. The classroom teacher shares with you that she thinks Rosalia may have an articulation disorder, but the teacher is not sure. The teacher provides you with some examples of things that Rosalia has said in the past 2 or 3 weeks. As you look at these examples, which one of the following would not be typical for her in terms of predictable productions based on Spanish influence? A. Omission of /h/ in word-initial position (e.g., -elp/help) B. w/r substitutions (e.g., wing/ring) C. t/th substitutions in word-initial positions (e.g., tin/thin) D. Devoicing of final consonants (e.g., beece/bees) E. Insertion of the schwa before word-initial s-clusters (e.g., esleep/sleep)

B. w/r substitutions (e.g., wing/ring)

44. You just completed an assessment of an 8-year-old boy who stutters. When you are offering post assessment counseling to the boy's parents, they ask you, "What do you think caused stuttering in our son?" How would you answer their question? Select the best among the alternatives given. A. "Although we have many theories, we don't know the cause of stuttering because no theory has been substantiated." B. "Stuttering is a genetically inherited disorder in almost all cases because more males than females stutter; there is a certain concordance rate for identical twins." C. "We can't say for sure in individual cases, but both complex genetic susceptibility and environmental factors may be involved in its causation." D. "Research suggests that stuttering is caused by parental pressure on the child to be more fluent; it is usually associated with high demands for fluency." E. "Research on the laryngeal mechanism suggests that a defect in the neural control of the larynx causes stuttering."

C. "We can't say for sure in individual cases, but both complex genetic susceptibility and environmental factors may be involved in its causation."

2. The components of the vertebral column consists of A. 8 cervical, 10 thoracic, and 4 lumbar, 4 sacral, and 2-3 coccygeal (fused) vertebrae B. 9 cervical, 11 thoracic, and 3 lumbar, 7 sacral, and 5-6 coccygeal (fused) vertebrae C. 7 cervical, 12 thoracic, and 5 lumbar, 5 sacral, and 3-4 coccygeal (fused) vertebrae D. 10 cervical, 111 thoracic, and 6 lumbar, 6 sacral, and 4-5 coccygeal (fused) vertebrae E. 11 cervical, 11 thoracic, and 3 lumbar, 3 sacral, and 1-2 coccygeal (fused) vertebrae

C. 7 cervical, 12 thoracic, and 5 lumbar, 5 sacral, and 3-4 coccygeal (fused) vertebrae

104. You have been asked to give an in-service to a group of students who wish to eventually specialize in service delivery to children with cleft palates and their families. The students want to know detailed information about in utero development of the hard and soft palate (among other things). You can accurately tell them that in utero the hard palate fuses between which developmental ages? A. 1-2 weeks B. 4-6 weeks C. 8-9 weeks D. 10-12 weeks E. 14-18 weeks

C. 8-9 weeks

28. In assessing children with language disorders, clinicians often count the number of morphemes in a child's utterance. This counting helps assess the length of single utterances the child typically produces. Clinicians follow certain rules in counting morphemes. Of the following rules, which one is correct? A. Inflected words, such as looked, are counted as one morpheme. B. When words are repeated for emphasis (e.g., "No, no, no!"), each repetition is counted as a morpheme (i.e., three morphemes in the example). C. A compound word, such as birthday, is counted as one morpheme. D. Contracted negatives (e.g., won't, can't) are counted as two morphemes. E. Uh and um are each counted as one morpheme.

C. A compound word, such as birthday, is counted as one morpheme.

107. A school-based clinician is assessing the velopharyngeal adequacy of Angelina, a 14-year-old immigrant high school student from Bali. Angelina was born with a cleft palate and lip; there was no repair until Angelina's family came to the United States when Angelina was 12 years old. In Bali, Angelina and her family lived in a small and rural area where surgery was unavailable. Though the repair surgery in the United States a year ago was successful and Angelina now has a more aesthetically pleasing appearance and better speech, there is still audible nasal emission and hypernasality when she speaks. The clinician plans to refer Angelina to a local craniofacial team, but he still wants to conduct as thorough an examination as he can despite the lack of instrumentation available at his school site. The clinician does have access to an oral manometer. He uses his examination at the school site to provide a beginning point from which to refer Angelina to the craniofacial team. After obtaining a ratio by comparing pressures achieved in the nostrils-occluded and the nostrils-open conditions, the clinician concludes that Angelina needs to be referred to the craniofacial team for possible further surgery or a pharyngeal flap. When he did oral manometry, what did the clinician probably find that Angelina had? A. A ratio of 1.0 B. A ratio of 1.2 C. A ratio of .87 D. A ratio of .91 E. A ratio of .98

C. A ratio of .87

80. A patient comes to you stating that she cannot hear well on the telephone anymore and has resorted almost exclusively to texting as a means of communicating with her children and friends. She avoids parties and restaurants as well as other noisy situations because it is difficult to understand what others are saying. She has her hearing tested, and the resulting audiogram has the configuration below. What can you assume that this patient has? A. Central auditory processing disorder B. A mixed hearing loss C. A sensorineural hearing loss D. A conductive hearing loss E. An acoustic neuroma

C. A sensorineural hearing loss

77. A person with otosclerosis often has an audiogram reflecting Carhart's notch. What is Carhart's notch? A. Specific losses at both 2000 and 4000 Hz as indicated by bone-conduction testing B. A specific type of sensorineural hearing loss characterized by a "dip" at 1000 Hz C. A specific loss at 2000 Hz as indicated by bone-conduction testing D. A specific loss at 4000 Hz as indicated by both air- and bone-conduction testing E. A specific loss at 2000 Hz as indicated by air-conduction testing

C. A specific loss at 2000 Hz as indicated by bone-conduction testing

110. A child is referred to a hospital-based clinician for assessment and treatment. In the child's chart, it is stated that the child has a syndrome caused by spontaneous autosomal dominant mutations. The gene and the locus of this syndrome is FGR2 at 10q25-26. The child has midfacial hypoplasia, an arched and grooved hard palate, and mild mental retardation. What does the child have? A. Trisomy 13 B. Angelman syndrome C. Apert syndrome D. Turner syndrome E. Treacher-Collins syndrome

C. Apert syndrome

75. A fifth-grade teacher refers Bora to you. Bora speaks Thai. She and her family have been living in the United States for 2 years, and Bora has been enrolled in U.S. schools for that whole time. However, she has often been sick and has missed many days of school. The family is helpful, and Bora's parents do their best to work on assignments with her at home. However, their conversational English is limited, and they do not read or write in English at all. When you talk to the teacher, he states that Bora has friends and gets along well in the classroom. However, she especially struggles in the area of reading. The school team meets and decides to use an RtI (response to intervention) approach to discern whether Bora has a language and experiential difference or a language-learning disability. Which one of the following is in the RtI model? A. The classroom teacher will continue "business as usual," providing no extra or additional instruction and assessing whether Bora makes progress. B. Bora will automatically undergo an extensive special education evaluation that will determine her possible need for placement in a self-contained special education classroom. C. Bora's teacher will implement the use of scientifically based instruction in the regular education setting to provide her with additional reading support; if this is insufficient to improve her performance, the special education team will evaluate her for possible special education services. D. Bora will automatically undergo special education evaluation that will determine her possible need for pull-out speech-language therapy and academic support from the resource specialist. E. Bora will be given additional homework packets to complete during the after-school homework program. Instructions will be written in Thai and English so that the Thai-speaking interpreter can complete the homework with Bora in a timely fashion and to the teacher's satisfaction.

C. Bora's teacher will implement the use of scientifically based instruction in the regular education setting to provide her with additional reading support; if this is insufficient to improve her performance, the special education team will evaluate her for possible special education services.

7. Various arteries help supply blood to the face and the brain. Neurogenic communication disorders are associated with interrupted blood supply to the brain. Of the following statements about the arteries that supply blood to the brain, which one is correct? A. Internal carotid artery supplies the muscles of the face. B. Damage to the external carotid artery causes aphasia. C. Broca's area and Wernicke's area are supplied by the middle cerebral artery. D. If an artery below the circle of Willis is blocked, the brain damage is maximal. E. The anterior cerebral artery primarily supplies the occipital lobe.

C. Broca's area and Wernicke's area are supplied by the middle cerebral artery.

92. As a clinician, you are concerned with using the most appropriate assessment approach that suits your clients, avoids false positive or false negative diagnoses, helps generate treatment goals, and is fair to clients of all ethnocultural backgrounds, including mainstream clients. To accomplish this assessment goal, you would select which of the following approaches? A. Portfolio approach B. Authentic approach C. Criterion-referenced approach D. Integrated approach E. Dynamic approach

C. Criterion-referenced approach

101. A researcher wishes to assess the efficacy of the new Riverton method of training children to say /r/ accurately. He decides to use a single-subject design because he has a private practice where he serves a number of children who have w/r substitutions. The researcher needs to keep several concepts in mind as he begins his research. Which one of the following concepts is false? A. An example of a multiple-baselines-across-settings design would be teaching a behavior (e.g., correct /r/ production) sequentially in different settings to demonstrate that the behavior changes only in a treated setting and thus treatment is effective. B. In a multiple-baselines-across-subjects design, several subjects are taught a behavior sequentially to show that only treated subjects change and thus that the treatment is effective. C. In the ABAB withdrawal design, a target behavior is base rated (e.g., /r/ in the A phase), taught to the subject (e.g., accurately produced /r/ in the B phase), reduced by teaching its counterpart or an incompatible behavior (e.g., teaching w/r in the A phase), and then taught again (e.g., accurately produced /r/ in the B phase) to show that the treatment is effective. D. ABAB designs rely upon two conditions: (a) the A phase, or no-treatment phase, where target behaviors are base rated with no treatment; and (b) the B phase, or treatment phase, where the target behavior is treated. E. For this researcher, an advantage of using a single-subject design to evaluate the efficacy of the Riverton method of training children to say /r/ accurately is that he can integrate research and clinical service by using the clients he serves as subjects in an experiment that attempts to answer a significant clinical question.

C. In the ABAB withdrawal design, a target behavior is base rated (e.g., /r/ in the A phase), taught to the subject (e.g., accurately produced /r/ in the B phase), reduced by teaching its counterpart or an incompatible behavior (e.g., teaching w/r in the A phase), and then taught again (e.g., accurately produced /r/ in the B phase) to show that the treatment is effective.

86. Which of the following is a limitation of standardized speech-language tests? A. Generally exhaustive time required to administer them B. Lack of statistical norms C. Inadequate participant and response sampling D. Unnecessarily extensive testing of each individual skill sampled in the test E. Not being able to evaluate a client's performance with that of his or her peers

C. Inadequate participant and response sampling

95. Researchers often use cross-sectional studies to study children's development in various areas. What is a difficulty with cross-sectional studies? A. The same subjects are studied over time, and this is expensive, time consuming, and difficult because subjects might drop out of the study. B. The investigator is examining data already on file to answer questions about children in various age groups, and that data might not be reliable. C. Observations are made of differences between subjects of different ages to generalize about developmental changes that would occur within subjects as they mature. D. The total age span of children to be studied is divided into several overlapping age spans, and it is difficult to follow subjects from the lower to the upper end of each age span. E. Observations are made of differences within subject groups of different ages to generalize about developmental changes that would occur between subjects as they mature.

C. Observations are made of differences between subjects of different ages to generalize about developmental changes that would occur within subjects as they mature

120. A clinician in a private practice is approached by the parents of Cole, a 5-year-old boy. They want to place Cole in kindergarten in the fall, but they share that "we know there's something wrong with him—we're just not sure what." According to Cole's parents, he is a "sweet, lovable boy who will go to anybody. He likes to sing a lot, too." Because the parents live in a rural area, health-care access has been limited. After seeing Cole for the first time, the clinician refers his parents to a neurologist because she suspects that Cole has a syndrome. Cole is small for his age and has an elfin-like appearance characterized by a small chin, turned-up nose, puffiness around the eyes, a long upper lip, and a wide mouth. Cole's teeth are small and widely spaced. The clinician knows that she will probably end up seeing Cole for intervention if his parents are able to bring him on a weekly basis. The clinician will probably be working on which of the following goals? A. Morphological skills, because although children with this syndrome usually have above average IQs, they frequently delete bound morphemes from the beginnings and ends of words B. Feeding, because children with this syndrome tend to have feeding problems C. Overall expressive and receptive language, because children with this syndrome generally have IQs between 50 and 70, although some of these children have good language skills D. Oral motor coordination, because children with this syndrome usually have oral-motor coordination problems that contribute to decreased intelligibility E. Pragmatics, to increase Cole's ability to interact with others

C. Overall expressive and receptive language, because children with this syndrome generally have IQs between 50 and 70, although some of these children have good language skills

10. Dynamics of speech production involve several important concepts. Select the statement that accurately describes one of the concepts of speech production dynamics. A. Phonetic adaptations refer to the influence of adjacent sounds on speech sound production. B. Assimilation refers to changes in vocal tract configurations. C. Phonetic adaptations refer to the variations in articulatory movements and changes in vocal tract configurations. D. Assimilation refers to the variations in articulatory movements. E. Coarticulation is not related to assimilation or adaptation.

C. Phonetic adaptations refer to the variations in articulatory movements and changes in vocal tract configurations.

24. You are developing a language treatment program for a child diagnosed with Asperger's syndrome. In your treatment program, you include which of the following main and initial skills? A. Morphologic language skills B. Syntactic language skills C. Pragmatic language skills, including social appropriateness D. Increased verbalizations on topics of their interest E. Monologue as a means of teaching longer different types of sentences

C. Pragmatic language skills, including social appropriateness

59. Select the statement that is not true. A. Dysarthria and Broca's aphasia may coexist. B. Excessive or even stress on syllables is not a part of ataxic dysarthria. C. Roughly 94% of Parkinson's patients have Hypokinetic dysarthria. D. Spastic-ataxic and flaccid-spastic are frequently grouped in the mixed variety of dysarthria. E. Monopitch and monoloudness are not found in patients with ataxic dysarthria.

C. Roughly 94% of Parkinson's patients have Hypokinetic dysarthria.

55. In a comprehensive oral examination for a Master's degree, a faculty member describes a 40-year-old adult woman who talks to herself, uses confused language (although the speech is mostly grammatical), and describes events and experiences that may be unreal. The student is asked to diagnose the communication problem of the woman described. Which of the following is the correct answer the student should give? A. Global aphasia B. Wernicke's aphasia C. Schizophrenia D. Apraxia of speech E. Traumatic brain injury

C. Schizophrenia

91. While working in a school setting, you are planning to assess a child whose parents speak Spanish. The child's primary language is Spanish, although the child is in the process of learning English. You want to find out if any standardized test will be applicable to the child. In selecting a particular test, you should consider which of the following? A. You can have a family member translate English test items into Spanish. B. You can go ahead and administer an English test because the child is learning English anyway. C. Spanish-speaking children of the same background as the client have been adequately sampled in the standardization process of a potentially useful test. D. You should consider whether you can translate English test items into Spanish. E. You should consider whether you can modify each English test item to suit the child.

C. Spanish-speaking children of the same background as the client have been adequately sampled in the standardization process of a potentially useful test.

20. You have been asked to assess the language skills of 7-year-old Max, who was referred by his classroom teacher. The teacher says that "Max talks in these really short sentences, and he doesn't talk very much. I dont know if he is just shy or if there is more going on. He does seem to understand what I say, though." The teacher is concerned because in her second-grade class she has worked on oral-language skills daily. It is close to the end of the year, and the teacher is concerned about how Max will perform in third grade. You decide to conduct an informal language screening to decide whether you need to formally evaluate Max's expressive-language skills. You find he uses sentences such as "He has a ball" and "I like Pokémon." Max uses few complex or compound sentences. His parents say this behavior is also typical at home. What is your next step? A. Inform the teacher and parents that Max might have autistic-like tendencies and that he needs to be formally evaluated by his pediatrician and the school psychologist. B. Tell the teacher and parents that Max is within normal limits for his age and that a formal language evaluation is unnecessary. C. Tell the teacher and parents that you would like to formally evaluate Max's language skills because, at 7 years of age, he should be using compound and complex sentences, and his language should approximate the adult model. D. Tell the teacher and parents that you will take a "wait and see" approach. If the third-grade teacher has concerns similar to those of the second-grade teacher, you will follow up with a formal evaluation of Max's language skills. E. Immediately place Max into therapy based on these screening results because he clearly has a language delay and you want to begin therapy immediately.

C. Tell the teacher and parents that you would like to formally evaluate Max's language skills because, at 7 years of age, he should be using compound and complex sentences, and his language should approximate the adult model.

54. What does the cover-body theory of phonation state? A. The superficial layer of the lamina propria and much of the intermediate layer of the lamina propria vibrate as a "cover" on a relatively stationary "body," which is made up of the remainder of the intermediate layer, the deep layer, and the TA muscle. B. The epithelium, the superficial layer of the lamina propria, and much of the intermediate layer of the lamina propria vibrate as a "cover" on a relatively stationary "body," which is made up of the remainder of the intermediate layer and the deep layer. C. The epithelium, the superficial layer of the lamina propria, and much of the intermediate layer of the lamina propria vibrate as a "cover" on a relatively stationary "body," which is made up of the remainder of the intermediate layer, the deep layer, and the TA muscle. D. The epithelium and much of the intermediate layer of the lamina propria vibrate as a "cover" on a relatively stationary "body," which is made up of the remainder of the intermediate layer and the TA muscle. E. The epithelium, the deep layer of the lamina propria, and much of the superficial layer of the lamina propria vibrate as a "cover" on a relatively stationary "body," which is made up of the remainder of the superficial layer, the deep layer, and the TA muscle.

C. The epithelium, the superficial layer of the lamina propria, and much of the intermediate layer of the lamina propria vibrate as a "cover" on a relatively stationary "body," which is made up of the remainder of the intermediate layer, the deep layer, and the TA muscle.

84. What are speech reception thresholds (SRTs)? A. The highest level of hearing at which a person can understand 50% of the words presented B. Determined by looking at the frequencies most important to speech in the patient's pure-tone air conduction test results C. The lowest level of hearing at which a person can understand 50% of the words presented D. The lowest level of hearing at which a person can understand 100% of the words presented E. Determined by the patient's response to a list of monosyllabic words presented at 30 dB HL

C. The lowest level of hearing at which a person can understand 50% of the words presented

66. You are watching an experienced clinician conduct swallowing therapy for a 68-year-old woman with dysphagia. You note that the clinician teaches the patient to tilt her head forward while swallowing and to switch between liquid and semisolid swallows, and applies gentle pressure on one side of the thyroid cartilage during the swallow. This kind of treatment is appropriate for the disorders of which of the following phases? A. The oral preparatory phase B. The oral phase C. The pharyngeal phase D. The esophageal phase E. The oral-esophageal phase

C. The pharyngeal phase

41. You have been referred an 8-year-old boy because the teacher suspects that he stutters. Before rendering a diagnosis, you wish to determine the different kinds of dysfluencies he exhibits. You have taken an extended speech sample, and you are now counting the different forms of dysfluencies. Select the following statement that is correct in measuring the types and the number of dysfluencies in specified utterances. A. The utterance "I went-went-went to uh p-p-play videogames with my ffffriend on SSSSS well um Sa-Sa-Sa-Saturday" contains three word repetitions, two interjections, and two-syllable prolongations. B. The utterance "I-I-I could not do-do-do it because I was af-af-afraid of it" contains two sound repetitions and one word repetition. C. The utterance "I wanted to-I wanted to-I wanted to ssssssee what um my mmmmmom was d-d-d-doing," contains one phrase repetition, two sound prolongations, one sound interjection, and one part-word (sound) repetition. D. The utterance "He-he-he didn't tell me anything be[silence]fore he [silence] before he d-d—decided to do it" contains one word repetition, two pauses, and one sound repetition. E. The utterance "Let me tell you that during the winter break . . . lllllet me um tell well t-t-tell you . . . never mind!" contains one incomplete sentence and two sound prolongations.

C. The utterance "I wanted to-I wanted to-I wanted to ssssssee what um my mmmmmom was d-d-d-doing," contains one phrase repetition, two sound prolongations, one sound interjection, and one part-word (sound) repetition.

37. Employees at an infant-care program ask you for advice about a baby who has been placed in their care. They say the baby is "playing" with his speech mechanism and producing noises such as raspberries, squeals, yells, and some CV-like combinations. The baby is 5 months old. What do you tell the personnel? A. This is not normal; the baby should be in the variegated babbling stage of development. B. This is not normal; the baby should be in the cooing or gooing stage of development. C. This is normal; the baby is appropriately in the expansion stage of development. D. This is normal; the baby is in the reduplicated babbling stage of development. E. This is not normal; the baby should be in the expanded intonational stage of development.

C. This is normal; the baby is appropriately in the expansion stage of development.

33. If you use phonetic placement to help a child produce /r/, what will you do? A. Draw the child's attention to the internal structure of words that contain /r/ B. Incorporate rhyming words containing /r/ to increase the child's metalinguistic awareness skills C. Use verbal instructions, modeling, physical guidance, and visual feedback to show the child how the /r/ is produced D. Emphasize auditory training at increasingly complex levels, using auditory bombardment and minimal pairs E. Begin at the syllable level and introduce /r/ in various contexts, emphasizing principles of coarticulation

C. Use verbal instructions, modeling, physical guidance, and visual feedback to show the child how the /r/ is produced

106. What is the surgical method of cleft palate repair that involves raising two bipedicled flaps of mucoperiosteum, bringing them together, and attaching them to close the cleft? A. Veau-Wardill-Kilner method B. V-Y retroposition C. Von Langenbeck surgical method D. Pharyngeal flap procedure E. Palatoplasty

C. Von Langenbeck surgical method

68. A teacher has referred Jeremy to you for an evaluation. Jeremy is an African American first grader who is reportedly doing well in class academically. When you observe him on the playground with his peers, you see that he has many friends and does not appear to have problems interacting appropriately with other children. Jeremy's friends do not appear to have any difficulty understanding what he says. However, the teacher is concerned because she says, "I think Jeremy pronounces some of his sounds wrong. I think he needs speech therapy." When you conduct a speech screening with Jeremy, you will remember that which one of the following patterns indicates a possible disorder, not a difference, based on Jeremy's use of African American English (AAE)? A. f/th substitution in word-final position B. Production of [ks] instead of [sk] C. th/s substitution in all word positions D. Differing stress on some words (e.g., police instead of police) E. b/v substitution in all word positions

C. th/s substitution in all word positions

16. What would a child whose utterance contained an embedded form say? A. "My dad drove me to school, and he dropped me off at the car-pool place." B. "Because I did my homework right after school, I got to play extra Game Boy." C. "My mom took away my cell phone because I didn't get a good report card." D. "The teacher who gave a lot of homework wasn't very popular." E. "We wanted to have a party; however, we had a school project to finish."

D. "The teacher who gave a lot of homework wasn't very popular."

42. Although differences of opinion exist, some clinicians diagnose stuttering on the basis of the frequency of dysfluencies when all forms of dysfluencies are counted in a speech sample. If you accept this practice, you would then diagnose stuttering if the dysfluency rate in a speech sample is A. 10% or more B. 3.5% or more C. 15% or more D. 5% or more E. 7.5% or more

D. 5% or more

14. As you analyze a language sample, you find an utterance made by the child: "big pizza." This utterance is an example of which type of semantic relation? A. Action + locative B. Agent + action C. Attribute + locative D. Attribute + entity E. Possession + attribute

D. Attribute + entity

105. A child comes to your clinic with her mother for articulation and language therapy. The mother tells you that her daughter has Hurler's syndrome. What causes Hurler's syndrome? A. An expanded number of CGG nucleic acid repeats on a specific gene on one of the distal ends of the Y chromosome B. A spontaneous autosomal dominant mutation of FGR2 at 10q25-26 C. Autosomal dominant inheritance and deletion in the region of the long arm of chromosome 15 (15q11-15q13) D. Autosomal recessive deficiency of X-L iduronidase E. An extra whole number chromosome 22, resulting in 47, rather than the normal 46 chromosomes

D. Autosomal recessive deficiency of X-L iduronidase

57. You are assessing a 60-year-old woman for a possible diagnosis of aphasia, and you are interested in the specific type of aphasia this woman might have. You notice that the woman has marked buccofacial apraxia. This condition alone might suggest which of the following? A. Wernicke's aphasia B. Anomic aphasia C. Transcortical sensory aphasia D. Broca's aphasia, transcortical motor aphasia, or conduction aphasia E. Conduction aphasia

D. Broca's aphasia, transcortical motor aphasia, or conduction aphasia

46. A 12-year-old boy has been referred to you with a note from the teacher that the boy speaks very fast and his speech is difficult to understand. The teacher believes the boy may have an articulation disorder. Your assessment suggests extremely rapid speech, compression of syllables, deletion of syllables in words, transposition of sounds in words, and a high rate of dysfluencies, but normal articulation of sounds at a slower speech rate. Your diagnosis of this boy should be which of the following? A. A complex cognitive communication disorder B. Neurogenic fluency disorder C. A severe phonological disorder D. Cluttering E. Apraxia of speech

D. Cluttering

88. To select an appropriate language production test for a 5-year-old male child, you examine several test manuals. You come across a test manual that claims that the test meets the theoretical expectation that sampled language skills are higher at progressively higher age levels. You then correctly conclude that the manual claims that the test has which of the following? A. Content validity B. Acceptable reliability C. Consistency of scores at progressively higher age levels D. Construct validity E. Predictive validity

D. Construct validity

32. A 15-year-old high school sophomore with th/s substitution comes to you for therapy. She is frustrated because she wants to act in high school plays but has been told she cannot do this because of her "speech problem." She is highly motivated to produce /s/ correctly, and you begin seeing her for therapy. If you are using Van Riper's approach, what will you do? A. Begin therapy at the syllable level, focusing especially on principles of coarticulation as you help her produce /s/ accurately in various contexts B. Use a training approach that emphasizes three phases: establishment, transfer, and maintenance and that heavily emphasizes imitation C. Focus on a paired stimuli approach D. Focus on phonetic placement, auditory discrimination, and drill-like practice at increasingly complex motor levels until accurate /s/ production is automatized E. Use a cycles approach in which you do not drill /s/ to mastery but rather introduce the correct way to produce /s/, give the student limited practice producing /s/, and use auditory bombardment

D. Focus on phonetic placement, auditory discrimination, and drill-like practice at increasingly complex motor levels until accurate /s/ production is automatized

23. The criteria for diagnosing autism in children include which of the following? A. Normal intelligence, normal early language acquisition that begins to deteriorate, and appearance of symptoms after age 6 B. An unusual interest in acquiring nonverbal means of communication, a strong preference for varied environmental arrangements, and a lower than normal prevalence of seizure disorders C. Maternal failure to bond with the child, absence of motor deficits, absence of pragmatic language deficiencies D. Impaired social interaction, stereotypic behaviors and interests, and disturbed communication E. Rapid overgeneralization of words, lack of social tact, and absence of sensory deficits

D. Impaired social interaction, stereotypic behaviors and interests, and disturbed communication

51. You have been asked to counsel with John, a 70-year-old man who has smoked and drunk alcohol since he was a teenager. He now has laryngeal cancer, and, before surgery, the surgeon asks you to talk with John about esophageal speech. You explain to John that there are two basic types of esophageal speech. In one method, the patient is taught to keep the esophagus open and relaxed while inhaling rapidly. In the other method, the patient impounds the air in the oral cavity, pushes it back into the esophagus, and vibrates the cricopharyngeus muscle. What is the second method called? A. Inhalation method B. Laryngeal airway resistance method C. Inhalatory injection method D. Injection method E. Injection resistance method

D. Injection method

116. As a clinician in a medically based private practice, you receive a referral of 37-year-old Jan, who has been a physical education teacher for the last 15 years. Jan works part time as a telemarketer, and, according to her husband, she "is glued to her cell phone." Jan also sings in the church choir. She has been hoarse for several years and tells you during the case history that "I've ignored the way I sound—it's just me. I haven't felt like I've needed to change anything." However, Jan shares that lately she has been feeling a lot of pain, and the hoarseness is substantially worse. She says, "Sometimes when I talk, it's almost like there's a 'double voice.'" Jan tells you she is worried because her job depends upon being able to yell and raise her voice on the playground as she teaches physical education. She is worried that she might lose her job if she can no longer yell. You immediately refer Jan to an otolaryngologist for a thorough examination of her vocal folds. You then proceed to do your own perceptual and instrumental evaluation. You arrive at a number of findings, including the fact that Jan has increased laryngeal dysphonia, airway resistance, and a maximum phonation time of 7 seconds. You think that Jan is a probable candidate for phonosurgery, but you will wait for the otolaryngologist's diagnosis and recommendations. Measures of jitter and shimmer are becoming more common with voice patients because such measures can be useful in early detection of vocal pathology. Although you suspect that the otolaryngologist will find obvious vocal pathology, given Jan's history of prolonged hoarseness, you still want to obtain measures of jitter and shimmer because these can serve as an excellent baseline—especially if Jan has phonosurgery. When you take these measures, what might you expect to see? A. Large amounts of both jitter and shimmer, with more than 1 dB of variation across vibratory cycles when jitter is measured B. Large amounts of both jitter and shimmer, with Jan being able to sustain a vowel with approximately 15% shimmer C. A small amount of shimmer and a large amount of jitter D. Large amounts of both jitter and shimmer, with more than 1 dB of variation across vibratory cycles when shimmer is measured E. A large amount of jitter, with only a small-to-moderate amount of shimmer

D. Large amounts of both jitter and shimmer, with more than 1 dB of variation across vibratory cycles when shimmer is measured

27. You are evaluating a 5-year-old boy whose mother abused alcohol while carrying her son. In your assessment, you would look for specific speech and language problems; in addition, you would also look for which of the following positive or negative signs? A. Normal motor and intellectual development, normal play activities, and normal facial and skull features B. Language problems, but no speech problems, coupled with good eye contact and generally compliant behavior C. Lack of gestures, good eye contact, and lack of attachment to new people D. Low birth weight and length, behavior problems, and possible swallowing difficulties E. Lack of separation anxiety, passive interaction with peers, and good short-term memory

D. Low birth weight and length, behavior problems, and possible swallowing difficulties

69. A classroom teacher refers a 10-year-old African American male student to you because the teacher is concerned about his intelligibility. This teacher is anxious to avoid the mistake of mislabeling this student as having a "speech disorder" if he is merely manifesting characteristics of AAE. When you screen the boy, you find that he makes the following substitutions: d/m, f/n, and m/n. What would you do? A. Provide therapy for the student because, even though these are typical patterns for speakers of AAE, the boy needs to learn Mainstream American English articulation patterns B. Do nothing, knowing that boys mature more slowly than girls do C. Do nothing, realizing that this is typical for speakers of African American English D. Provide intervention for the student, because this is a sign of a speech sound disorder involving substitutions of nasals with other sounds E. Let the classroom teacher address these issues in the classroom setting because this is such a mild problem

D. Provide intervention for the student, because this is a sign of a speech sound disorder involving substitutions of nasals with other sounds

38. A mother brings her 3-year-old daughter, Shannon, to you. The mother is a pediatrician and is worried that Shannon's "sounds are not developing the way they should. I wonder if she needs speech therapy." What can you tell this mother? A. Shannon should be producing consonant clusters with 80% to 90% accuracy. B. Shannon should definitely be producing /r/, /l/, /th/, and /s/ accurately. C. Shannon should have mastered /f/ but will still be developing /p/, /m/, and /n/. D. Shannon should be in the process of mastering glides /w/ and /j/, but liquids /r/ and /l/ will probably develop later. E. Shannon should be in the process of acquiring consonant clusters but will still have difficulty producing /f/.

D. Shannon should be in the process of mastering glides /w/ and /j/, but liquids /r/ and /l/ will probably develop later.

94. An investigator carries out a study in which the effect of rate of speech upon stuttering during sibling interaction is being investigated. The investigator gathers conversational samples from children who stutter and their siblings. In the control group, siblings are asked to speak as they normally would at home. In the experimental group, siblings are asked to speak much more quickly than they would at home. The investigator wishes to measure the rate of siblings' speech upon the amount of stuttering done by the children who stutter. In other words, the investigator is asking if increased rate of siblings' speech causes children to stutter more. In this study, what is the dependent variable? A. The amount of stuttering done by children who stutter when siblings speak at a slowed rate B. The rate of speech of the siblings in the experimental group C. The rate of speech of the siblings in the control group D. The amount of stuttering done by the children who stutter when the siblings increase their rate of speech E. The combined amount of stuttering done by the children in both the experimental and the control groups

D. The amount of stuttering done by the children who stutter when the siblings increase their rate of speech

96. A researcher teaches a new book-reading program to caregivers of low-income children and evaluates the language skills of the children before the implementation of the program and one year later. The researcher's goal is to evaluate whether there is a relationship between caregivers' implementation of the program and children's growth in language skills. The researcher finds that there is an r = .13 correlational relationship between caregivers' reported implementation of the program and children's language skills. What can the researcher safely conclude? A. There is a mildly significant cause-and-effect relationship between the caregivers' implementation of the program and children's language skills. B. There is a strong negative correlation between the caregivers' implementation of the program and children's language skills. C. There is a moderately significant cause-and-effect relationship between the caregivers' implementation of the program and children's language skills. D. There is no significant relationship between the caregivers' implementation of the program and children's language skills. E. There is a strong positive correlation between the caregivers' implementation of the program and children's language skills.

D. There is no significant relationship between the caregivers' implementation of the program and children's language skills.

63. In treating the communication deficits of a young adult with traumatic brain injury, you would do which of the following? A. Refrain from using techniques to increase orientation and attention because of the inappropriateness of these treatment targets B. Exclusively use cognitive rehabilitation C. Introduce a variety of treatment activities to promote faster recovery D. Withhold attention from irrelevant and inappropriate responses E. Never use tangible reinforcers because they are unnatural

D. Withhold attention from irrelevant and inappropriate responses

6. The different parts of the brain are connected by bundles of fibers. Therefore, the brain functions as an integrated whole. Of those connecting fibers, the projection fibers A. are interhemispheric connectors B. consist of superior longitudinal fibers C. do not contain motor fibers D. connect the cortex and the subcortical structures E. consist of only efferent fibers

D. connect the cortex and the subcortical structures

11. Being a branch of physics, acoustics describes various physical properties of sound, its generation, and propagation. Of the following descriptions, which one is incorrect? A. Newton's law of inertia states that all bodies always remain at rest until a force sets them into motion. B. The process of a sound wave traveling back after hitting an obstacle with no change in its speed is called refraction. C. Formant frequency is a frequency region with concentrated energy. D. Measured in terms of Nt, force is the product of mass and acceleration. E. A sinusoidal wave contains a single frequency. 12. Sound is defined as vibrations or disturbances of molecules in a medium. This means that A. the sound waves actually travel from point A to point B B. sound waves are always audible; otherwise we could not study them C. a rigid medium can be as good as a flexible medium in transmitting sound D. sound waves are disturbances in adjacent molecules within a medium E. vibrations that produce sound always repeat themselves at regular intervals

D. sound waves are disturbances in adjacent molecules within a medium

79. Key parts of the auditory nervous system include cranial nerve VIII, which has two branches: the __________ branch and the __________ branch, which carries the electrical sound impulses from the cochlea to the brain. A. retrocochlear, auditory B. cochlear, auditory-acoustic C. auditory-acoustic, retrocochlear D. vestibular, auditory-acoustic E. retrocochlear, vestibular

D. vestibular, auditory-acoustic

35. You are working closely with an orthodontist who frequently refers children to your private practice. Many of these children have protrusion of the maxilla and retrusion of the mandible accompanied by a condition in which the upper teeth from the molars forward are positioned excessively anterior to the lower teeth. What do these children have? A. A class I malocclusion accompanied by underjet B. A class II malocclusion accompanied by underjet C. A class III malocclusion accompanied by overjet D. A class III malocclusion accompanied by underjet E. A class II malocclusion accompanied by overjet

E. A class II malocclusion accompanied by overjet

117. As a clinician in a medically based private practice, you receive a referral of 37-year-old Jan, who has been a physical education teacher for the last 15 years. Jan works part time as a telemarketer, and, according to her husband, she "is glued to her cell phone." Jan also sings in the church choir. She has been hoarse for several years and tells you during the case history that "I've ignored the way I sound—it's just me. I haven't felt like I've needed to change anything." However, Jan shares that lately she has been feeling a lot of pain, and the hoarseness is substantially worse. She says, "Sometimes when I talk, it's almost like there's a 'double voice.'" Jan tells you she is worried because her job depends upon being able to yell and raise her voice on the playground as she teaches physical education. She is worried that she might lose her job if she can no longer yell. You immediately refer Jan to an otolaryngologist for a thorough examination of her vocal folds. You then proceed to do your own perceptual and instrumental evaluation. You arrive at a number of findings, including the fact that Jan has increased laryngeal dysphonia, airway resistance, and a maximum phonation time of 7 seconds. You think that Jan is a probable candidate for phonosurgery, but you will wait for the otolaryngologist's diagnosis and recommendations. You receive a phone call and a report from the otolaryngologist that Jan will indeed need phonosurgery for the presence of bilateral vocal fold polyps (the right polyp is larger than the left). The otolaryngologist wants you to obtain quantitative measurements of Jan's voice before phonosurgery; he wants to use these baseline measures as a comparison with measures taken after phonosurgery to evaluate whether the phonosurgery was successful. To obtain these quantitative measurements, what will you probably use? A. A plethysmograph B. Indirect or mirror laryngoscopy C. A Nasometer D. Esophageal manometry E. A sound spectrograph

E. A sound spectrograph

53. A client comes to you seeking voice therapy. Samantha is a 31-year-old male transgender client who has undergone several gender reassignment procedures to become more feminine. She tells you that she is also taking estrogen. Samantha shares that she needs help to speak in a more feminine way, but she does not know how to go about this. She is also dealing with emotional issues surrounding her gender reassignment. In this case, what ideally should you do? A. Share with Samantha that it is ideal to have a combination of surgery and voice therapy to teach such feminine communication patterns as increased pitch, increased upward inflection at the ends of utterances, and female body language B. Tell Samantha that various surgical procedures, such as thyroplasty, are available and that having surgical procedures will be sufficient to help her change her voice to sound more feminine C. Advise Samantha that surgical procedures are unnecessary, but that voice therapy will help her to sound more feminine by teaching new communication patterns, such as higher pitch and more feminine intonation patterns D. Advise Samantha that a combination of voice therapy and counseling will be the best way for her to sound more feminine and also receive emotional support as she deals with gender reassignment issues E. Advise Samantha that a combination of counseling, surgery, and voice therapy to teach more feminine pitch levels and communication patterns would best serve her needs

E. Advise Samantha that a combination of counseling, surgery, and voice therapy to teach more feminine pitch levels and communication patterns would best serve her needs

58. You are evaluating a 65-year-old man who has been referred to you by a neurologist. The client's report suggests a history of a single left hemisphere stroke with no clinically significant muscle paralysis or weakness. Your orofacial examination reveals a possible reduction in oral sensation. Speech evaluation suggests that the client makes an effort to slow down his speech, but his speech is full of variable articulatory breakdowns on repeated attempts at producing the same word. You also find anticipatory substitutions, metathetic errors, easier automatic production than volitional production, and failed attempts at self-correction. Based on these findings, which of the following is your diagnosis? A. Right hemisphere syndrome B. Unilateral upper motor neuron dysarthria C. Broca's aphasia D. Alzheimer's dementia E. Apraxia of speech

E. Apraxia of speech

103. A hospital-based clinician is conducting an experiment with patients with aphasia. She is assessing the efficacy of a new therapy kit, The Basic Object Memory Builder (The BOMB), in increasing the memory skills of her patients. The experimental and control groups have been carefully matched on all variables. Halfway through the experiment, the clinician finds that many of the experimental subjects go to a local senior citizens' center several times a week for Bingo and other games. There are refreshments afterward, and a group of high school student volunteers comes and acts as conversational partners with the subjects. None of the control subjects go to the senior citizens' center. At the end of the experiment, the clinician finds that the subjects in the experimental group who were treated using The BOMB have improved significantly in their memory skills compared to the control subjects, who have been treated with more traditional memory therapy techniques. What can the researcher safely conclude from her study? A. The BOMB was more successful in helping the experimental patients improve their word memory skills than were the traditional methods in helping patients in the control group improve their memory skills. B. The BOMB was not successful in helping the patients in the experimental group improve their word-retrieval skills; the improvement was due to weekly attendance at the senior citizens' center. C. Traditional memory therapy techniques are the most reliable and valid; the clinician should not attempt to use The BOMB again. D. The BOMB was only moderately successful in helping experimental group patients improve their memory skills. E. Because most experimental subjects attended weekly Bingo nights at the senior citizens' center was a possible confounding variable, making it impossible to conclude that The BOMB alone caused the difference in the performance of the two groups.

E. Because most experimental subjects attended weekly Bingo nights at the senior citizens' center was a possible confounding variable, making it impossible to conclude that The BOMB alone caused the difference in the performance of the two groups.

73. You are a monolingual English-speaking speech-language pathologist working in a Head Start program in a city with many children from culturally and linguistically diverse (CLD) backgrounds. In the last year, CLD children from more than 22 different cultural and linguistic groups have come to the Head Start speaking only their primary languages. What would be the optimal plan for helping these children succeed in Head Start and eventually in elementary school? A. Hire aides who speak only the children's primary languages and not English, so that the children will be in the ideal situation to maintain their primary languages and not really learn English until kindergarten B. Tolerate the children's use of their primary languages and speak to them solely in English, hoping they will eventually "pick up" English and discontinue using their primary languages C. Assume that these children probably have language-learning disabilities in their primary languages, and hire bilingual speech-language pathologists to assist in remediation D. Encourage them not to speak their primary languages and to speak English as much and as quickly as possible to avoid the inevitable confusion when young children speak more than one language fluently E. Hire bilingual aides from the neighboring communities and use their services to help the children learn English as well as maintain their primary languages.

E. Hire bilingual aides from the neighboring communities and use their services to help the children learn English as well as maintain their primary languages.

48. You are evaluating a 10-year-old boy who was referred because of difficulties associated with partial submucous cleft palate accompanied by a bifid uvula. He is being teased at school for "sounding funny," and his parents are concerned about how he talks. During your evaluation, what can you probably expect to find? A. Hypernasality, leading to difficulty producing nasals adequately B. Hyponasality, accompanied by increased intraoral breath pressure, leading to difficulties with adequate production of liquids and glides C. Intermittent hyponasality, leading to difficulty producing nasals adequately D. Intermittent hypernasality, accompanied by difficulty producing vowels and nasals adequately E. Hypernasality, accompanied by decreased intraoral breath pressure, leading to difficulties with adequate production of fricatives, affricates, and plosives

E. Hypernasality, accompanied by decreased intraoral breath pressure, leading to difficulties with adequate production of fricatives, affricates, and plosives

62. Critical diagnostic features of the right hemisphere syndrome include which of the following? A. Acceptance of illness B. Intact discourse skills C. Ease with expressing emotions that are experienced D. Intact prosody E. Impaired narrative skills

E. Impaired narrative skills

113. The Health Insurance and Portability and Accountability Act (HIPAA) passed by the U.S. Congress mandates certain rules that all health professionals must follow. Of the following statements regarding HIPAA, select the one that is correct. A. Consumers who think their privacy may have been violated by a medical provider can complain only to the provider in question. B. The law (HIPAA) does not provide for civil or criminal penalties for facilities that misuse patient information. C. HIPAA regulations apply to hospitals and medical doctors, not to other allied health professionals. D. HIPAA regulations do not cover electronically transmitted medical records of patients. E. Medical information about a patient may be shared by other professionals who serve the same patient, but it cannot be released for purposes unrelated to the care of the patient.

E. Medical information about a patient may be shared by other professionals who serve the same patient, but it cannot be released for purposes unrelated to the care of the patient.

17. A teacher has referred a second grader, Delanie, to you. As you assess Delanie's language, you see that she has difficulty with word endings. For example, she says things such as "My candy is tasty than yours" instead of "My candy is tastier than yours," or "He was happy than she was" instead of "He was happier than she was." Delanie is having difficulty specifically with which aspect of language? A. Pragmatics B. Emergent literacy C. Semantics D. Syntax E. Morphology

E. Morphology

81. A 76-year-old man with presbycusis comes to you complaining that when he is in social situations, such as church picnics, people do not speak loudly enough. He says that the noise creates a problem for him in hearing what people are saying. With what does this client have difficulty? A. Pragmatic skills B. Auditory memory C. Figure-ground discrimination D. Auditory discrimination E. Signal-to-noise ratio

E. Signal-to-noise ratio

83. What is the muscle that exerts the pull that allows the eustachian tube to open during yawning and swallowing? A. Levator veli palatini B. Stapedius muscle C. Tensor tympani D. Levator palatini E. Tensor palatini

E. Tensor palatini

89. You are planning treatment for a child who needs to learn morphological features of language. You have selected four grammatical morphemes to teach at the word level. You will then teach each word to a training criterion. You show a stimulus card, ask a question, wait for the response, reinforce the correct response, score the response, and wait for a few seconds before presenting the next opportunity for the child to produce the target morpheme. What is this procedure called? A. The shaping method B. The successive approximation C. The naturalistic method D. The nonstructured approach E. The discrete trial method

E. The discrete trial method

118. A clinician in a private practice is approached by the parents of Cole, a 5-year-old boy. They want to place Cole in kindergarten in the fall, but they share that "we know there's something wrong with him; we're just not sure what." According to Cole's parents, he is a "sweet, lovable boy who will go to anybody. He likes to sing a lot, too." Because the parents live in a rural area, health-care access has been limited. After seeing Cole for the first time, the clinician refers his parents to a neurologist because she suspects that Cole has a syndrome. Cole is small for his age and has an elfin-like appearance characterized by a small chin, turned-up nose, puffiness around the eyes, a long upper lip, and a wide mouth. Cole's teeth are small and widely spaced. What does the clinician suspect that Cole has? A. Hurler syndrome B. Apert syndrome C. Moebius syndrome D. Cri du chat syndrome E. Williams syndrome

E. Williams syndrome

8. Allophones A. change word meanings B. are not variations of phonemes C. are not perceived as the same D. may not vary from production to production E. may vary from production to production

E. may vary from production to production

100. A researcher is describing the speech of a group of young adults who have been diagnosed as clutterers. He finds that the faster they speak, the less intelligible they are. The researcher obtains a Pearson r correlational relationship of -.89. One could say that this shows _______ between rate of speech and intelligibility. A. virtually no correlational relationship B. a positive correlational relationship C. a multifactorial correlational relationship D. a cause-and-effect relationship E. strong negative correlational (or inverse) relationship

E. strong negative correlational (or inverse) relationship

4. The source-filter theory of speech production states that A. the larynx filters the sound before the sound travels through the vocal tract B. the resonating cavities of speech are the source of speech production C. the oral cavity is the source of all English sounds D. the nasal cavity is not important in speech production E. the laryngeally produced sound is modified by the resonating cavities

E. the laryngeally produced sound is modified by the resonating cavities

43. Stuttering is more common on A. vowels than consonants B. the second syllable than on the third or the subsequent syllables C. the second word in a sentence than the first D. the more frequently used words E. the less frequently used words

E. the less frequently used words


Conjuntos de estudio relacionados

Texas Principles of Real Estate 1: Chapter 8 Quiz

View Set

ATI Chapter 1 Maternal Newborn: Contraception

View Set